Proof by induction: n^3 < n for n >=6

  • Thread starter Thread starter 1MileCrash
  • Start date Start date
  • Tags Tags
    Induction Proof
Click For Summary
The discussion focuses on proving that n^3 < n! for all n >= 6 using mathematical induction. The base case for n = 6 is established as true. The inductive step involves assuming k^3 < k! and examining (k+1)^3, which can be expressed as k^3 + 3k^2 + 3k + 1. By applying the induction hypothesis and factoring, it is shown that the expression remains less than k!(k+1), thus completing the proof. The approach emphasizes the importance of bounding the expression effectively to validate the induction step.
1MileCrash
Messages
1,338
Reaction score
41

Homework Statement


Show that n^3 < n! for all n >= 6.

Homework Equations

The Attempt at a Solution



We see that for the base case of n = 6, the claim holds.

Suppose that k^3 < k! for some natural number k >= 6.

Consider that:
(k+1)^3
= k^3 + 3k^2 + 3k + 1
< k! + 3k^2 + 3k + 1 [By induction hypothesis]

What's a neat way to finish this? I'm a bit rusty, apparently.[/B]
 
Physics news on Phys.org
1MileCrash said:

Homework Statement


Show that n^3 < n! for all n >= 6.

Homework Equations

The Attempt at a Solution



We see that for the base case of n = 6, the claim holds.

Suppose that k^3 < k! for some natural number k >= 6.

Consider that:
(k+1)^3
= k^3 + 3k^2 + 3k + 1
< k! + 3k^2 + 3k + 1 [By induction hypothesis]

What's a neat way to finish this? I'm a bit rusty, apparently.[/B]

Factor a ##k^3## out of ##k^3+3k^2+3k+1##. Then use ##k^3 \le k!## and ##(k+1)!=k!(k+1)##.
 
Alright. And then, what remains after being factored has its largest value at k=6, and its value is smaller than any (k+1), and so I may write < k!(k+1), completing the induction.
 
1MileCrash said:
Alright. And then, what remains after being factored has its largest value at k=6, and its value is smaller than any (k+1), and so I may write < k!(k+1), completing the induction.

Sounds ok.
 
Question: A clock's minute hand has length 4 and its hour hand has length 3. What is the distance between the tips at the moment when it is increasing most rapidly?(Putnam Exam Question) Answer: Making assumption that both the hands moves at constant angular velocities, the answer is ## \sqrt{7} .## But don't you think this assumption is somewhat doubtful and wrong?

Similar threads

  • · Replies 15 ·
Replies
15
Views
3K
  • · Replies 3 ·
Replies
3
Views
2K
Replies
7
Views
4K
  • · Replies 1 ·
Replies
1
Views
4K
  • · Replies 11 ·
Replies
11
Views
3K
  • · Replies 6 ·
Replies
6
Views
2K
  • · Replies 7 ·
Replies
7
Views
993
Replies
3
Views
4K
  • · Replies 7 ·
Replies
7
Views
2K
  • · Replies 3 ·
Replies
3
Views
2K